Revision as of 19:10, 27 June 2012 by Hu45 (Talk | contribs)


ECE Ph.D. Qualifying Exam: Automatic Control (AC)- Question 3, August 2011

 $ \color{blue}\text{5. } \left( \text{20 pts} \right) \text{ Consider the optimization problem, } $

                            $ \text{optimize} \left(x_{1}-2\right)^{2}+\left(x_{2}-1\right)^{2} $

                        $ \text{subject to } x_{2}- x_{1}^{2}\geq0 $

                                                 $ 2-x_{1}-x_{2}\geq0, x_{1}\geq0. $

$ \color{blue} \text{The point } x^{*}=\begin{bmatrix} 0 & 0 \end{bmatrix}^{T} \text{ satisfies the KKT conditions.} $

$ \color{blue}\left( \text{i} \right) \text{Does } x^{*} \text{ satisfy the FONC for minimum or maximum? Where are the KKT multipliers?} $

$ \color{blue}\text{Solution 1:} $

$ \text{ Standard form: optimize} \left(x_{1}-2\right)^{2}+\left(x_{2}-1\right)^{2} $

$ \text{subject to } g_{1}\left( x \right) x_{1}^{2}-x_{2}\leq0 $

$ g_{2}\left( x \right) x_{1}+x_{2}-2\leq0 $

$ g_{3}\left( x \right) -x_{1}\leq0 $



$ \color{blue}\text{Solution 2:} $



Automatic Control (AC)- Question 3, August 2011
Problem 1:  https://www.projectrhea.org/rhea/index.php/ECE-QE_AC3-2011_solusion
Problem 2:  https://www.projectrhea.org/rhea/index.php/ECE-QE_AC3-2011_solusion-2
Problem 3:  https://www.projectrhea.org/rhea/index.php/ECE-QE_AC3-2011_solusion-3
Problem 4:  https://www.projectrhea.org/rhea/index.php/ECE-QE_AC3-2011_solusion-4


Back to ECE Qualifying Exams (QE) page

Alumni Liaison

Abstract algebra continues the conceptual developments of linear algebra, on an even grander scale.

Dr. Paul Garrett